Is there a proportional relationship between x and y? Explain.

Is There A Proportional Relationship Between X And Y? Explain.

Answers

Answer 1

Answer:

c

Step-by-step explanation:

14-11=3

12-9=3

10-7=3

8-5=3

Answer 2

Answer:

A

Step-by-step explanation:

It isn’t proportional because when you graph the coordinates, it doesn’t pass the origin, so it isn’t equivalent. Therefore, making the answer A.


Related Questions

how to find the volume of an oven with the inside dimensions 3 ft by 1.5 ft by 3 ft

Answers

9514 1404 393

Answer:

  13.5 ft³

Step-by-step explanation:

The volume of a cuboid is the product of its length, width, and height:

  V = (3 ft)(1.5 ft)(3 ft) = 13.5 ft³

The volume of the oven is 13.5 ft³.

__

You may see this as the formula ...

  V = LWH

geomtry plz help 15 points

Answers

Answer:

m∠O = 41°

Step-by-step explanation:

∠NOM=∠NMO=(4y-15)° (base angles of isos triangle)

7y+2(4y-15)=180 (angle sum of triangle)

  7y+8y-30=180

       15y-30=180

             15y=180+30

                  =210

                y=210÷15

                  =14

Hence, m∠O = (4y-15)°

                      = [4(14)-15]°

                      = (56-15)°

                      = 41°

Alf needs to borrow $15,000 to pay for his college tuition. He can borrow the money from his
parents at a rate of 3.55% interest compounded annually for 4 years, or he can borrow from his
local bank at a rate of 3.50% interest compounded continuously for 4 years
Manipulation of Numerical Data

Answers

Alf would prefer to borrow from his parents because their rates are more favourable.

The future value of the loan if he borrows from his parents is given by this formula: A(1 + r)^n

Where:

A = amount  

R = interest rate  

N = number of years  

$15,000(1 + 0.0355)^4 = $17,246.13

The future value of the loan if he borrows from the local bank is given by this formula: : A x e^r x N

Where:

A= amount e = 2.7182818 N = number of years r = interest rate

$15,000 x  2.7182818^0.035 x 4 = $62,137.18

To learn more about future value, please check: https://brainly.com/question/14640433

What is a quartic function with only two real zeros at x = 7 and x = 13
A. y = (x - 10) ^ 5 - 81 OR y = x ^ 5 - 20x ^ 4 + 92x ^ 3 + 34x ^ 2 - 20x + 91

B. Y= (x - 10) ^ 4 - 81 OR y = x^4 - 20x +91

C. y = (x - 3) ^ 4 - 41 OR y = 2x ^ 4 - 21x ^ 3 + 52x ^ 2 - 17x + 609

D. y = (x+ 8)^4 + 71 OR y = x^4 - 3x^3 + 7x^2 - 5x + 13

Answers

Zeros be a and b

a=7b=13

Function

[tex]\\ \sf\longmapsto y=x^2-(a+b)x+ab[/tex]

[tex]\\ \sf\longmapsto y=x^2-(7+13)x+(7)(13)[/tex]

[tex]\\ \sf\longmapsto y=x^2-20x+91[/tex]

Option B

g A long-term study has revealed that a test for cancer in men is very effective. The study shows that 89% of the men for which the test is positive actually have cancer. If a man selected at random tests positive for cancer with this test, what is the probability that he does not have cancer

Answers

The answer is 89% because I had that Same question and it was 89%

Suppose a jar contains 17 red marbles and 32 blue marbles. If you reach in the jar and pull out 2 marbles at random, find the probability that both are red.

Answers

There are 49 total marbles, the chance of you getting a red is 17/49, and if you don’t replace the marble and pull out another one, the probability of that being red is 17/49*16/48=17/147. That’s approximately a .12% chance.
16/48=1/3

The probability that both are red marbles = [tex]\bold{\frac{17}{147} }[/tex]

What is probability?

"It is finding out the possibilities of the occurrence of an event."

Formula to find the probability of an event:

"P(A) = n(A) / n(S)

where, n(A) is the number of favorable outcomes of an event A

n(S) is the total number of outcomes for an experiment"

For given example,

A jar contains 17 red marbles and 32 blue marbles.

If we pull out 2 marbles at random, we need to find the probability that both are red.

The number of possible outcomes,

[tex]\Rightarrow n(S)=^{49}C_2\\\\ \Rightarrow n(S)=\frac{49!}{2!(49-2)!}\\\\ \Rightarrow n(S)=1176[/tex]

The number of possible outcomes of selecting both the red marbles.

[tex]\Rightarrow n(A)=^{17}C_2\\\\ \Rightarrow n(A)=\frac{17!}{2!(17-2)!}\\\\ \Rightarrow n(A)=136[/tex]

The probability that both the marbles are red,

[tex]\Rightarrow P(A)=\frac{n(A)}{n(S)}\\\\ \Rightarrow P(A)=\frac{136}{1176}\\\\ \Rightarrow P(A)=\frac{17}{147}[/tex]

Therefore, the probability that both are red marbles = [tex]\bold{\frac{17}{147} }[/tex]

Learn more about the probability here:

brainly.com/question/11234923

#SPJ2

What is the coefficient of x in expression of 5mx​

Answers

Coefficient of this expression is 5 only.

- BRAINLIEST answerer

Find the value of X that makes lines L and M parallel.

Answers

Answer:

x = 11

Step-by-step explanation:

    If the lines were parallel, then the transversal would make 2x + 1 and 23° alternate interior angles. Alternate interior angles equal one another, so set up the following equation.

[tex]2x+1=23~(Given)\\\\2x+1-1=23-1~(Subract~1~on~both~sides)\\\\2x=22~(Simplify)\\\\\frac{2x}{2}=\frac{22}{2}~(Divide~2~on~both~sides)\\\\x=11~(Simplify)[/tex]

lol mine looks different


[tex]2 {}^{3} \times 2 - {}^{3} [/tex]
I need answers

Answers

The answer is 13 I might be wrong but I think it’s 13
The answer is 13

Step by step

2^3 x 2 - 3=
2 x2 x2=8
8 x2=16
16-3=13

geomtry plzzz help 15 points

Answers

Answer:

19

Step-by-step explanation:

they give you xz, and they want half, so just divide 38 by 2

Write the equation of the function graphed below:

a. f(x) = 2x−−√3 –4
b. f(x) = 2x−−√3 + 4
c. f(x) =x−−√3 + 4
d. f(x) =x−−√3 – 4

Answers

Answer:

I have a couple questions before I answer, what do the 2 dashes represent and are the 4s also under the square root?

In a school there are 200 boys and 800 girls. Find the percentage of girls in that school.

Answers

Answer:

80% girls.

Step-by-step explanation:

20% boy.

80%+20%= 100%

Juan wants to buy a video game for $63. He saves $12 every Friday. Part A. Create an equation to represent Juan's total savings, y, in dollars, after x Fridays. y​

Answers

Answer:

Answer in explanation.

Step-by-step explanation:

An equation to represent Juan's total savings y, in dollars, after x Fridays will be;

⇒ y = 12x

What is an expression?

Mathematical expression is defined as the collection of the numbers variables and functions by using operations like addition, subtraction, multiplication, and division.

Given that;

Juan wants to buy a video game for $63.

And, He saves $12 every Friday.

Now,

Let total dollars after x Friday = y

And, He saves $12 every Friday.

So, We can formulate;

⇒ Total money (y) = 12x

⇒ y = 12x

Thus, An equation to represent Juan's total savings y, in dollars, after x Fridays will be;

⇒ y = 12x

Learn more about the mathematical expression visit:

brainly.com/question/1859113

#SPJ2

The width of a triangle is six more than twice the height. The area of the triangle is 88in2. Find the height and width of the triangle.

Answers

Answer:

w = 22 in, h = 8 in.

Step-by-step explanation:

Reading the problem, I immediately see two equations. This hints me towards a system of equations problem.

Statement 1 is The width of a triangle is six more than twice the height.

The width (w) of a triangle is (=) six (6) more (+) than twice (2) the height (h).

Equation 1: w = 6 + 2h

Statement 2 is The area of the triangle is 88 in2.

The formula for the area of a triangle is [tex]\frac{wh}{2}[/tex]. So, we get our second equation.

Equation 2: wh/2 = 88

We see that we have the same number of equations as unknowns, two. This is important as it lets us know that we can find two unknowns in the two equations. If there are more unknowns than equations, then we cannot find the values of all the variables. Look for another equation in the problem. However, if the # equations greater than or equal to number of unknowns, then we are ready to start solving!  So, we can now move to solving.

Equation 1 isolates one of the unknowns (w). So, we can replace w in the second equation with the right side of eq 1. Then, we simplify for h.

[tex]\frac{wh}{2} = 88[/tex]

[tex]\frac{(6 + 2h)h}{2} = 88[/tex]

[tex]\frac{6h + 2h^2}{2} = 88[/tex]

[tex]6h + 2h^2 = 176[/tex]

[tex]2h^2 + 6h - 176 = 0[/tex]

[tex]h = -11, 8[/tex] (assuming you know how to solve quadratics)

Since we know that heights of triangles cannot be negative, we find that [tex]h = 8[/tex] in.

For the final step, we plug [tex]h[/tex] into our first equation to find [tex]w[/tex].

[tex]w = 6 + 2h[/tex]

[tex]w = 6 + 2(8)[/tex]

[tex]w = 6 + 16[/tex]

[tex]w = 22[/tex]

So, the answer is w = 22 in, h = 8 in.

To check if we were right, we can plug the values into both equations to see if they are equal. I will skip equation 1 since we used that equation "as-is" to find w.

[tex]\frac{wh}{2} = 88[/tex]

[tex]\frac{22 * 8}{2} = 88[/tex]

[tex]11 * 8 = 88[/tex]

[tex]88 = 88[/tex]

The solution checks out!

Evaluate this function: h(t) = |t+2| + 3; Find h(6)

Answers

Answer:

Step-by-step explanation:

h(6) = |6 + 2| + 3 = |8| + 3 = 8 + 3 = 11


A high school track team's long jump record is 22 ft 6 3/4in. This year, Arthur's best long jump is 21 ft 10 1/2in. If long jumps are measured to the nearest quarter inch, how much farther must
Arthur jump to break the record?
To beat the school long jump record,

Arthur must jump an additional ___inches.

(Type a whole number, fraction, or mixed number.)

Answers

Arthur must jump an additional 8.25 inches so as to break the record.

1 foot =  12 inches

Long jump record = 22 ft 6 3/4in = 22 ft 6.75 in = (22ft * 12 in per ft) + 6.75 in

Long jump record = 270.75 in

Arthur's long jump record = 21 ft 10 1/2in = 21 ft 10.5 in = (21ft * 12 in per ft) + 10.5 in

Arthur's long jump record = 262.5 ft.

To break the record, the distance needed to be jump by Arthur = 270.75 in - 262.5 ft. = 8.25 in

Arthur must jump an additional 8.25 inches so as to break the record.

Find out more at: https://brainly.com/question/20796404

When it was 70 degrees outside. 50 members showed up at a beach club. For each degree the temperature rose, another 10 members came to the beach club. Write the function, Kt), that represents the number of members at the beach club as a function of the temperature​

Answers

Answer:

what to do here I don't know sorry

...............

.

.

........

.

.

...............

please help this is for my study guide thanks! ​

Answers

The volume is 29.32 <3

plz help on a timed test will pay 10 points and make brainlest

What other countries have set up early earthquake warning systems? Which of these countries are in the Ring of Fire? Why is it important for these Ring of Fire countries to have warning systems?

Answers

Answer:

Mexico, Japan, South Korea, Taiwan, and the United States, Chile, Japan, the US west coast;  Chile, Japan, the US west coast; So all that is left for the people who live around the Ring of Fire is to be aware of the danger, perhaps to live further inland, build safer, earthquake-resistant housing, and for nations everywhere to improve oceanic and land-based early-warning systems to help minimize the risk to life

Step-by-step explanation:

what is 4% of 32? is it a percent, base, or amount

Answers

Answer:

1.28 - amount

Step-by-step explanation:

You would multiply 32 times 0.04 and get 1.28 which I assume would be an amount.

The amount of 4% of 32 is 1.28.

We have to determine what is 4% of 32.

What is the percentage?

This free percentage calculator computes a number of values involving percentages, including the percentage difference between two given values.

Suppose it is an x percent.

Therefore x multiply 32 times 0.04 and get 1.28 an amount.

To learn more about the percentage of visits:

https://brainly.com/question/24304697

#SPJ2

Is 40 is 50% of 80 true or false
With step by step evidence

Answers

Answer:

please mark brainliest

it is true

Step-by-step explanation:

if you multiplie 80 by 0.50 you will get 40,

e/kaid_274404147475337871026587/assig!
Khan Academy
Percent word problems
Of the 50 U.S. states, 4 have names that start with the letter W.
What percentage of U.S. states have names that start with the letter W?
0
Stuck? Review related articles/videos or use a hint.

Answers

Answer:

Step-by-step explanation:

4/50 = 0.08

0.08(100) = 8%

How many pounds are in 1 1/2 pounds and 8 ounces? There are _____ pounds in 1 1/2 pounds and 8 ounces.

Answers

Answer:

Hope this helps

Step-by-step explanation:

Since 16 ounces equal 1 pound, we’ll add 16 + 16, which is 32 ounces. For one half of a pound, simply divide the amount of ounces in a pound (16) by half. 16 divided by 2 is 8, so in total your answer would be 40 ounces in 2 1/2 pounds.

Solve the equation: 3.017 + k = 5.134

Answers

Answer:

k=2.117

Step-by-step explanation:

Answer:

3.017 + k = 5.134

So, k = 5.134 - 3.017

So, k = 2.117

see question above ^^​

Answers

I think the answer is (4) because if you rotate AEC to the left (counterclockwise) and follow by the scale factor of 2 so, CE = 4x2 = 8 and CA = 3x2 = 6. Hope this help :))

35. In her rough work, Janu added 2389 and 740 as shown here.
Is it okay to write the number 740 this way in the addition?
(Hint: You do NOT need to calculate)
(A) Yes, because 0 has no value so 74 is the same as 740.
(B) No, because the answer will be wrong if 0 is not written.
(C) No, because the digits 7 and 4 have NOT been arranged according to their value.
(D) Yes, because 7 is added with the hundred's digit and 4 with the ten's digit.

Answers

Answer:

D

Step-by-step explanation:

Note: Enter your answer and show all the steps that you use to solve this problem in the space provided.

You buy clothing at a sale. You buy a sweater at
7
10
of its original price of $35. Answer each question and show all your work.

a. How much money did you spend?
b. How much money did you save?
c. What fraction of the total original price did you save?

Answers

The amount that will be spent is $24.50, the amount saved will be $10.50, and the percentage saved will be 30%.

The amount that will be spent will be:

= 7/10 × $35 = $24.50

The amount that will be saved will be:

= $35 - $24.50 = $10.50

The fraction of the original price that is saved will be:

= 10.50/35 × 100

= 30%

Read related link on:

https://brainly.com/question/25763817

What’s the slope of 10,6 and 4, 1.2

Answers

let's firstly change the 1.2 to a fraction

[tex]1.\underline{2}\implies \cfrac{12}{1\underline{0}}\implies \cfrac{6}{5} \\\\[-0.35em] ~\dotfill\\\\ (\stackrel{x_1}{10}~,~\stackrel{y_1}{6})\qquad (\stackrel{x_2}{4}~,~\stackrel{y_2}{\frac{6}{5}}) \\\\\\ \stackrel{slope}{m}\implies \cfrac{\stackrel{rise} {\stackrel{y_2}{\frac{6}{5}}-\stackrel{y1}{6}}}{\underset{run} {\underset{x_2}{4}-\underset{x_1}{10}}}\implies \cfrac{~~ \frac{6-30}{5}~~}{-6}\implies \cfrac{~~ \frac{-24}{5}~~}{-6}\implies \cfrac{~~ -\frac{24}{5}~~}{-\frac{6}{1}}[/tex]

[tex]-\cfrac{\stackrel{4}{~~\begin{matrix} 24 \\[-0.7em]\cline{1-1}\\[-5pt]\end{matrix}~~}}{5}\cdot -\cfrac{1}{\underset{1}{~~\begin{matrix} 6 \\[-0.7em]\cline{1-1}\\[-5pt]\end{matrix}~~}}\implies \boxed{\cfrac{4}{5}}[/tex]

La probabilidad de que julio salga con carla es 0.75 , y la probabilidad de que salga con marisol es de 0.50. Si la probabilidad de que salga con carla o marisol es 0.85; calcular la probabilidad de que salga con ambas a la vez

Answers

La probabilidad de que Julio salga con Marisol y Carla al mismo tiempo es 0.40

What is the addition rule of probability for two events?

For two events A and B, we have:

Probability that event A or B occurs = Probability that event A occurs + Probability that event B occurs - Probability that both the event A and B occur simultaneously.

This can be written symbolically as:

[tex]P(A \cup B) = P(A) + P(B) - P(A \cap B)[/tex]

Given that;

The probability that Julio goes out with Carla is 0.75, and

the probability that he goes out with Marisol is 0.50.

The probability that he goes out with Carla or Marisol is 0.85

To find:

The probability that he will date both at the same time.

Let we take:

A = event that Julio goes with Carla

B = event that Julio goes with Marisol, then, the given expression is symbolically represented as:

[tex]P(A) = 0.75\\[/tex][tex]P(B) = 0.50[/tex][tex]P(A \cup B) = 0.85[/tex]

To find: [tex]P(A \cap B)\\[/tex]

Using the addition rule, we get:

[tex]P(A \cup B) = P(A) + P(B) - P(A \cap B)\\0.85 = 0.75 + 0.50 - P(A \cap B)\\P(A \cap B) = 1.25 - 0.85 = 0.40[/tex]

Thus, the probability that Julio will date Marisol and Carla at the same time is 0.40

Learn more about probability here:

brainly.com/question/1210781

The probability that Julio will date Marisol and Carla at the same time is 0.40.

Here, we have, For two events A and B, we have:

Probability that event A or B occurs = Probability that event A occurs + Probability that event B occurs - Probability that both the event A and B occur simultaneously.

This can be written symbolically as:

P(A ∪ B) = P(A) + P(B) - P(A ∩ B)

Given that;

The probability that Julio goes out with Carla is 0.75, and

the probability that he goes out with Marisol is 0.50.

The probability that he goes out with Carla or Marisol is 0.85

To find:

The probability that he will date both at the same time.

Let we take:

A = event that Julio goes with Carla

B = event that Julio goes with Marisol, then, the given expression is symbolically represented as:

P(A) = 0.75

P(B) = 0.50

P(A∪B)= 0.85

To find: P(A ∩ B)

Using the addition rule, we get:

P(A ∪ B) = P(A) + P(B) - P(A ∩ B)

0.85 = 0.75 + 0.50 - P(A ∩ B)

P(A ∩ B) = 0.40

Thus, the probability that Julio will date Marisol and Carla at the same time is 0.40.

Learn more about probability here:

brainly.com/question/1210781

#SPJ3

complete question:

The probability that Julio dates Carla is 0.75, and the probability that he dates Marisol is 0.50. If the probability that he goes out with carla or marisol is 0.85; Calculate the probability that he will date both at the same time.

31 divided by 943.89

Answers

Answer:

0.032

Step-by-step explanation:

Answer:  0.03284281007

Step-by-step explanation:

Other Questions
Round 18.527 to the nearest tenth. Which of the following is likely to happen if you leave backface culling on when you are working on an object? What did Lizzie do when she "putone foot forward"? A retirement account was opened with a $900 deposit. If the account earns 4.25% interest compounded continuously and has no other deposits or withdrawals, how long will it take the value to double? In _____ personality disorder, the affected person is aggressive and ruthless, and shows no sign of the conscience that would inhibit wrongdoing. How old was John F Kennedy when he died 100 points if you get this button. Tests are automatically saved in case of internet disruption.Language: English Espaol6)A student observes a cell under a microscope. The cell contains chromosomes that show up inside of a nucleus surrounded by anenvelope. Based on the infon nation here, the student was able to determine whether the cell was prokaryotic or eukaryotic. Aftermaking the distinction, the student could assume the cell came from certain cell types. From which potential organism(s) was thiscell-derived? Select ALL that apply.A)plant cellB)animal cellfungal cellD)bacterial cellE)methanogenic Archaea cell2 What is the mean absolute deviation (MAD) of the data set? 2, 6, 8, 12, 12Enter your answer as a decimal in the box. Help me please How can we have confidence in the white people? We have good and just reasons to believe we have ample grounds to accuse the Americans of injustice, especially when such great acts of injustice have been committed by them upon our race, of which they seem to have no manner of regard, or even to reflect. Everything I have told you is the truth. The Great Spirit has inspired me. ayuda es para hoy aaaa Zimbardo's simulation of prison life provided a dramatic demonstration of the power of _____ on behavior. PLEASE ANSWER NEED HELP!!!!!!!! PLEASE THE CORRECT ANSWER!!!!!!Zhiyu wants to start an exercise program. He knows that he should talk to his doctor, but he cannot get an appointment for several weeks. He does not have any health conditions. What is the BEST advice for Zhiyu?A. All exercise needs to be medically supervised, so he must wait.B. There is always a risk of injury, so he needs professional advice before he starts.C. He can start slowly with low impact activity like walking or light weightlifting.D. He should do whatever he feels like; his body will let him know if he overdoes it.PLEASE NO LINKS OR I WILL REPORT YOU At each step of the food chain, how much energy is used by the organism and how much is stored in their tissues? Find the solution set of the inequality143x Tristan wraps some gifts and then brings them to the post office where they are delivered to people in different parts of the country. Which organelle is Tristan most like? research on nonverbal communication indicates that Which of the following will require most developed countries to decrease emissions of carbon dioxide and other greenhouse gases? a. Montreal Protocol c. Emissions Reduction Act b. Kyoto Protocol d. Reforestation Project A car is travelling at a constant speed until the cars brakes are applied. The cars speed changes at a rate given by -0.08t ms^{-2} after the brakes are applied, where t s is the time since the brakes were applied. 3 seconds after the brakes are applied, the speed of the car is 5 ms^{-1} . How far will the car travel with the brakes applied before it stops? i just need help on plants